2.99 See Answer

Question: In Sect, 6.2.1, we began


In Sect, 6.2.1, we began with the potential of a perfect dipole (Eq. 6.10), whereas in fact we are dealing with physical dipoles. Show, by the method of Sect. 4.2.3, that we nonetheless get the correct macroscopic field.


> Try to compute the self-inductance of the “hairpin” loop shown in Fig. 7.38. (Neglect the contribution from the ends; most of the flux comes from the long straight section.) You’ll run

> Find the self-inductance per unit length of a long solenoid, of radius R, carrying n turns per unit length.

> A square loop of wire, of side a, lies midway between two long wires, 3a apart, and in the same plane. (Actually, the long wires are sides of a large rectangular loop, but the short ends are so far away that they can be neglected.) A clock- wise current

> A small loop of wire (radius a) is held a distance z above the center of a large loop (radius b), as shown in Fig. 7.37. The planes of the two loops are parallel, and perpendicular to the common axis. (a) Suppose current I flows in the big loop. Find the

> Imagine a uniform magnetic field, pointing in the z direction and filling all space / A positive charge is at rest, at the origin. Now somebody turns off the magnetic field, thereby inducing an electric field. In what direction does the charge move?16

> Where is ∂B/∂t nonzero, in Figure 7.21(b)? Exploit the analogy between Faraday’s law and Ampère’s law to sketch (qualitatively) the electric field

> Test the divergence theorem for the function v = (xy) xˆ + (2yz) yˆ+ (3zx) zˆ. Take as your volume the cube shown in Fig. 1.30, with sides of length 2.

> A capacitor C has been charged up to potential V0; at time t=0, it is connected to a resistor R, and begins to discharge (Fig. 7.5a). (a) Determine the charge on the capacitor as a function of time, Q(t). What is the current through the resistor, I (t)?

> A toroidal coil has a rectangular cross section, with inner radius a, outer radius a+w, and height h. It carries a total of N tightly wound turns, and the current is increasing at a constant rate (dI/dt=k). If w and h are both much less than a, find the e

> A square loop, side a, resistance R, lies a distance s from an infinite straight wire that carries current I (Fig. 7.29). Now someone cuts the wire, so I drops to zero. In what direction does the induced current in the square loop ï&n

> A long solenoid of radius a, carrying n turns per unit length, is looped by a wire with resistance R, as shown in Fig. 7.28. (a) If the current in the solenoid is increasing at a constant rate (dI/dt= k), what current flows in the loop, an

> An alternating current I=I0 cos (ωt) flows down a long straight wire, and returns along a coaxial conducting tube of radius a. (a) In what direction does the induced electric field point (radial, circumferential, or longitudinal)? (b) Assuming that the fiel

> A long solenoid with radius a and n turns per unit length carries a time-dependent current I (t) in the φˆ direction. Find the electric field (magnitude and direction) at a distance s from the axis (both inside and outside the solenoid), in the quasistati

> As a lecture demonstration a short cylindrical bar magnet is dropped down a vertical aluminum pipe of slightly larger diameter, about 2 meters long. It takes several seconds to emerge at the bottom, whereas an otherwise identical piece of unmagnetized ir

> A square loop of wire, with sides of length a, lies in the first quadrant of the xy plane, with one corner at the origin. In this region, there is a nonuniform time-dependent magnetic field / (where k is a constant). Find the emf induced in the loop.

> A long solenoid, of radius a, is driven by an alternating current, so that the field inside is sinusoidal: / A circular loop of wire, of radius a/2 and resistance R, is placed inside the solenoid, and coaxial with it. Find the current induced in the loop

> A square loop is cut out of a thick sheet of aluminum. It is then placed so that the top portion is in a uniform magnetic field B, and is allowed to fall under gravity (Fig. 7.20). (In the diagram, shading indicates the fiel

> Check the fundamental theorem for gradients, using T = x 2 + 4xy + 2yz3, the points a = (0, 0, 0), b = (1, 1, 1), and the three paths in Fig. 1.28: (a) (0, 0, 0) → (1, 0, 0) → (1, 1, 0) → (1, 1, 1); (

> A square loop (side a) is mounted on a vertical shaft and rotated at angular velocity ω (Fig. 7.19). A uniform magnetic field B points to the right. Find the E(t) for this alternating current generator.

> Two concentric metal spherical shells, of radius a and b, respectively, are separated by weakly conducting material of conductivity σ (Fig. 7.4a). (a) If they are maintained at a potential difference V, what current flows from o

> A short circular cylinder of radius a and length L carries a “frozen-in” uniform magnetization M parallel to its axis. Find the bound current, and sketch the magnetic field of the cylinder. (Make three sketches: one for L ((a, one for L

> A long circular cylinder of radius R carries a magnetization M = ks2 φˆ, where k is a constant, s is the distance from the axis, and φˆ is the usual azimuthal unit vector (Fig. 6.13). Find the

> An infinitely long circular cylinder carries a uniform magnetization M parallel to its axis. Find the magnetic field (due to M) inside and outside the cylinder.

> Of the following materials, which would you expect to be paramagnetic and which diamagnetic: aluminum, copper, copper chloride (CuCl2), carbon, lead, nitrogen (N2), salt (NaCl), sodium, sulfur, water? (Actually, copper is slightly diamagnetic; otherwise

> A uniform current density J = J0 zˆ fills a slab straddling the yz plane, from x = −a to x = +a. A magnetic dipole m = m0 xˆ is situated at the origin. (a) Find the force on the dipole, using Eq. 6.3. (b) Do the same for a dipole pointing in the y directi

> Derive Eq. 6.3. [Here’s one way to do it: Assume the dipole is an infinitesimal square, of side ε (if it’s not, chop it up into squares, and apply the argument to each one). Choose axes as s

> Find the force of attraction between two magnetic dipoles, m1 and m2, oriented as shown in Fig. 6.7, a distance r apart, (a) using Eq. 6.2, and (b) using Eq. 6.3.

> You are asked to referee a grant application, which proposes to determine whether the magnetization of iron is due to “Ampère” dipoles (current loops) or “Gilbert” dipoles (separated magnetic monopoles). The experiment will involve a cylinder of iron (ra

> Calculate the volume integral of the function T z2 over the tetrahedron with corners at (0,0,0), (1,0,0), (0,1,0), and (0,0,1).

> A magnetic dipole m is imbedded at the center of a sphere (radius R) of linear magnetic material (permeability μ). Show that the magnetic field inside the sphere (0 < r ≤ R) is / What is the field outside the sphere?

> At the interface between one linear magnetic material and another, the magnetic &iuml;&not;&#129;eld lines bend (Fig. 6.32). Show that tan &Icirc;&cedil;2/ tan &Icirc;&cedil;1 &Icirc;&frac14;2/&Icirc;&frac14;1, assuming there is no free current at the

> Compare Eqs. 2.15, 4.9, and 6.11. Notice that if &Iuml;&#129;, P, and M are uniform, the same integral is involved in all three: Therefore, if you happen to know the electric &iuml;&not;&#129;eld of a uniformly charged object, you can immediately write d

> Notice the following parallel: Thus, the transcription turns an electrostatic problem into an analogous magnetostatic one. Use this, together with your knowledge of the electrostatic results, to rederive (a) the magnetic &iuml;&not;&#129;eld inside a uni

> Imagine two charged magnetic dipoles (charge q, dipole moment m), constrained to move on the z axis (same as Problem 6.23(a), but without gravity). Electrically they repel, but magnetically (if both m’s point in the z direction) they attract. (a) Find th

> A familiar toy consists of donut-shaped permanent magnets (magnetization parallel to the axis), which slide frictionlessly on a vertical rod (Fig. 6.31). Treat the magnets as dipoles, with mass md and dipole moment m. (a) If you put two back-to-back magn

> In Prob. 6.4, you calculated the force on a dipole by &acirc;&#128;&#156;brute force.&acirc;&#128;&#157; Here&acirc;&#128;&#153;s a more elegant approach. First write B(r) as a Taylor expansion about the center of the loop: where r0 is the position of th

> (a) Show that the energy of a magnetic dipole in a magnetic &iuml;&not;&#129;eld B is [Assume that the magnitude of the dipole moment is &iuml;&not;&#129;xed, and all you have to do is move it into place and rotate it into its &iuml;&not;&#129;nal orient

> How would you go about demagnetizing a permanent magnet (such as the wrench we have been discussing, at point c in the hysteresis loop)? That is, how could you restore it to its original state, with M = 0 at I = 0?

> Starting from the Lorentz force law, in the form of Eq. 5.16, show that the torque on any steady current distribution (not just a square loop) in a uniform field B is m × B.

> Calculate the surface integral of the function in Ex. 1.7, over the bot- tom of the box. For consistency, let “upward” be the positive direction. Does the surface integral depend only on the boundary line for this function? What is the total flux over the

> Find the angle between the body diagonals of a cube.

> On the basis of the naïve model presented in Sect. 6.1.3, estimate the magnetic susceptibility of a diamagnetic metal such as copper. Compare your answer with the empirical value in Table 6.1, and comment on any discrepancy.

> A sphere of linear magnetic material is placed in an otherwise uniform magnetic field B0. Find the new field inside the sphere. [Hint: See Prob. 6.15 or Prob. 4.23.]

> A current I flows down a long straight wire of radius a. If the wire is made of linear material (copper, say, or aluminum) with susceptibility χm , and the current is distributed uniformly, what is the magnetic field a distance s from the axis? Find all th

> A coaxial cable consists of two very long cylindrical tubes, separated by linear insulating material of magnetic susceptibility &Iuml;&#135;m . A current I &iuml;&not;&#130;ows down the inner conductor and returns along the outer one; in each case, the c

> If J f=0 everywhere, the curl of H vanishes (Eq. 6.19), and we can express H as the gradient of a scalar potential W : H = −∇W. According to Eq. 6.23, then, ∇2 W = (∇ · M), so W obeys Poisson’s equation, with M as the “source.” This opens up all the mach

> For the bar magnet of Prob. 6.9, make careful sketches of M, B, and H, assuming L is about 2a. Compare Prob. 4.17.

> Suppose the &iuml;&not;&#129;eld inside a large piece of magnetic material is B0, so that H0 = (1/&Icirc;&frac14;0)B0 &acirc;&#136;&#146; M, where M is a &acirc;&#128;&#156;frozen-in&acirc;&#128;&#157; magnetization. (a) Now a small spherical cavity is h

> An infinitely long cylinder, of radius R, carries a “frozen-in” magnetization, parallel to the axis, M = ks zˆ, where k is a constant and s is the distance from the axis; there is no free current anywhere. Find the magnetic field inside and outside the cyl

> An iron rod of length L and square cross section (side a) is given a uniform longitudinal magnetization M, and then bent around into a circle with a narrow gap (width w), as shown in Fig. 6.14. Find the magnetic &iuml;&not;&#129;eld at the center of the

> Calculate the line integral of the function / from the origin to the point (1,1,1) by three different routes: (a) (0, 0, 0) → (1, 0, 0) → (1, 1, 0) → (1, 1, 1). (b) (0, 0, 0) → (0, 0, 1) → (0, 1, 1) → (1, 1, 1). (c) The direct straight line. (d) What is

> Calculate the torque exerted on the square loop shown in Fig. 6.6, due to the circular loop (assume r is much larger than a or b). If the square loop is free to rotate, what will its equilibrium orientation be?

> Find the magnetic &iuml;&not;&#129;eld at point P for each of the steady current con&iuml;&not;&#129;gurations shown in Fig. 5.23.

> (a) Find the magnetic &iuml;&not;&#129;eld at the center of a square loop, which carries a steady current I . Let R be the distance from center to side (Fig. 5.22). (b) Find the &iuml;&not;&#129;eld at the center of a regular n-sided polygon, carrying a

> For a con&iuml;&not;&#129;guration of charges and currents con&iuml;&not;&#129;ned within a volume V, show that where p is the total dipole moment. [Hint: evaluate /

> A thin glass rod of radius R and length L carries a uniform surface charge &Iuml;&#131; . It is set spinning about its axis, at an angular velocity &Iuml;&#137;. Find the magnetic &iuml;&not;&#129;eld at a distance s((R from the axis, in the xy plane (Fi

> Using Eq. 5.88, calculate the average magnetic &iuml;&not;&#129;eld of a dipole over a sphere of radius R centered at the origin. Do the angular integrals &iuml;&not;&#129;rst. Compare your answer with the general theorem in Prob. 5.59. Explain the discr

> A uniformly charged solid sphere of radius R carries a total charge Q, and is set spinning with angular velocity ω about the z axis. (a) What is the magnetic dipole moment of the sphere? (b) Find the average magnetic field within the sphere (see Prob. 5.5

> (a) A phonograph record carries a uniform density of “static electricity” σ . If it rotates at angular velocity ω, what is the surface current density K at a distance r from the center? (b) A uniformly charged solid sphere, of radius R and total charge Q

> (a) Prove that the average magnetic &iuml;&not;&#129;eld, over a sphere of radius R, due to steady currents inside / where m is the total dipole moment of the sphere. Contrast the electrostatic result, Eq. 3.105. [This is tough, so I&acirc;&#128;&#153;l

> A thin uniform donut, carrying charge Q and mass M , rotates about its axis as shown in Fig. 5.64. (a) Find the ratio of its magnetic dipole moment to its angular momentum. This is called the gyromagnetic ratio (or magnetomechanical ratio). (b) What is t

> Prove that the curl of a gradient is always zero. Check it for function (b) in Prob. 1.11.

> A magnetic dipole / is situated at the origin, in an otherwise uniform magnetic field / Show that there exists a spherical surface, centered at the origin, through which no magnetic field lines pass. Find the radius of this sphere, and sketch the field li

> Prove the following uniqueness theorem: If the current density J is specified throughout a volume V, and either the potential A or the magnetic field B is specified on the surface S bounding V, then the magnetic field itself is uniquely determined throughout

> Just as ∇ · B = 0 allows us to express B as the curl of a vector potential (B = ∇ × A), so ∇ · A = 0 permits us to write A itself as the curl of a “higher” potential: A = ∇ × W. (And this hierarchy can be extended ad infinitum.) (a) Find the general formu

> (a) Construct the scalar potential U(r) for a “pure” magnetic dipole m. (b) Construct a scalar potential for the spinning spherical shell (Ex. 5.11). [Hint: for r > R this is a pure dipole field, as you can see by comparing Eqs. 5.69 and 5.87.] (c) Try do

> Another way to &iuml;&not;&#129;ll in the &acirc;&#128;&#156;missing link&acirc;&#128;&#157; in Fig. 5.48 is to look for a magnetostatic analog to Eq. 2.21. The obvious candidate would be / (a) Test this formula for the simplest possible case&acirc;&#12

> (a) One way to &iuml;&not;&#129;ll in the &acirc;&#128;&#156;missing link&acirc;&#128;&#157; in Fig. 5.48 is to exploit the analogy between the de&iuml;&not;&#129;ning equations for and Maxwell&acirc;&#128;&#153;s equations for / Evidently A depends on

> Consider a plane loop of wire that carries a steady current I ; we want to calculate the magnetic &iuml;&not;&#129;eld at a point in the plane. We might as well take that point to be the origin (it could be inside or outside the loop). The shape of the w

> Magnetostatics treats the &acirc;&#128;&#156;source current&acirc;&#128;&#157; (the one that sets up the &iuml;&not;&#129;eld) and the &acirc;&#128;&#156;recipient current&acirc;&#128;&#157; (the one that experiences the force) so asymmetrically that it

> (a) If it is uniformly distributed over the surface, what is the surface current density K ? (b) If it is distributed in such a way that the volume current density is inversely proportional to the distance from the axis, what is J(s)?

> Suppose you wanted to &iuml;&not;&#129;nd the &iuml;&not;&#129;eld of a circular loop (Ex. 5.6) at a point r that is not directly above the center (Fig. 5.60). You might as well choose your axes so that r lies in the yz plane at (0, y, z). The source poi

> Prove that the divergence of a curl is always zero. Check it for function va in Prob. 1.15.

> Use Eq. 5.41 to obtain the magnetic field on the axis of the rotating disk in Prob. 5.37(a). Show that the dipole field (Eq. 5.88), with the dipole moment you found in Prob. 5.37, is a good approximation if z ((R.

> The magnetic field on the axis of a circular current loop (Eq. 5.41) is far from uniform (it falls off sharply with increasing z). You can produce a more nearly uniform field by using two such loops a distance d apart (Fig. 5.59). (a) Find the field (B) as

> Use the Biot-Savart law (most conveniently in the form of Eq. 5.42 appropriate to surface currents) to &iuml;&not;&#129;nd the &iuml;&not;&#129;eld inside and outside an in&iuml;&not;&#129;nitely long solenoid of radius R, with n turns per unit length, c

> Consider the motion of a particle with mass m and electric charge qe in the &iuml;&not;&#129;eld of a (hypothetical) stationary magnetic monopole qm at the origin: / (a) Find the acceleration of qe, expressing your answer in terms of q, qm , m, r (the p

> Calculate the magnetic force of attraction between the northern and southern hemispheres of a spinning charged spherical shell (Ex. 5.11). [Answer: (π/4)μ0σ 2ω2 R4.]

> A circularly symmetrical magnetic &iuml;&not;&#129;eld (B depends only on the distance from the axis), pointing perpendicular to the page, occupies the shaded region in Fig. 5.58. If the total &iuml;&not;&#130;ux / is zero, show that a charged particle

> A plane wire loop of irregular shape is situated so that part of it is in a uniform magnetic &iuml;&not;&#129;eld B (in Fig. 5.57 the &iuml;&not;&#129;eld occupies the shaded region, and points perpendicular to the plane of the loop). The loop carries a

> A current I &iuml;&not;&#130;ows to the right through a rectangular bar of conducting material, in the presence of a uniform magnetic &iuml;&not;&#129;eld B pointing out of the page (Fig. 5.56). (a) If the moving charges are positive, in which direction

> It may have occurred to you that since parallel currents attract, the current within a single wire should contract into a tiny concentrated stream along the axis. Yet in practice the current typically distributes itself quite uniformly over the wire. How

> Suppose that the magnetic field in some region has the form B = kz xˆ (where k is a constant). Find the force on a square loop (side a), lying in the yz plane and centered at the origin, if it carries a current I , flowing counterclockwise, when you look d

> Calculate the Laplacian of the following functions: (a) Ta = x 2 + 2xy + 3z + 4. (b) Tb = sin x sin y sin z. (c) Tc = e−5x sin 4y cos 3z. (d) v = x 2 xˆ + 3xz2 yˆ− 2xz zˆ.

> Analyze the motion of a particle (charge q, mass m) in the magnetic field of a long straight wire carrying a steady current I . (a) Is its kinetic energy conserved? (b) Find the force on the particle, in cylindrical coordinates, with I along the z axis. (

> I worked out the multipole expansion for the vector potential of a line current because that&acirc;&#128;&#153;s the most common type, and in some respects the easiest to handle. For a volume current J: (a) Write down the multipole expansion, analogous t

> (a) A phonograph record of radius R, carrying a uniform surface charge σ , is rotating at constant angular velocity ω. Find its magnetic dipole moment. (b) Find the magnetic dipole moment of the spinning spherical shell in Ex. 5.11. Show that for points

> Find the exact magnetic field a distance z above the center of a square loop of side w, carrying a current I . Verify that it reduces to the field of a dipole, with the appropriate dipole moment, when z ((w.

> A circular loop of wire, with radius R, lies in the xy plane (centered at the origin) and carries a current I running counterclockwise as viewed from the positive z axis. (a) What is its magnetic dipole moment? (b) What is the (approximate) magnetic field

> Show that the magnetic &iuml;&not;&#129;eld of a dipole can be written in coordinate- free form:

> Prove Eq. 5.78, using Eqs. 5.63, 5.76, and 5.77. [Suggestion: I’d set up Cartesian coordinates at the surface, with z perpendicular to the surface and x parallel to the current.]

> (a) Check Eq. 5.76 for the configuration in Ex. 5.9. (b) Check Eqs. 5.77 and 5.78 for the configuration in Ex. 5.11.

> (a) Complete the proof of Theorem 2, Sect. 1.6.2. That is, show that any diver genceless vector field F can be written as the curl of a vector potential A. What you have to do is find Ax , Ay , and Az such that (i) ∂ Az/∂ y ∂ Ay /∂ z Fx ; (ii) ∂ Ax /∂ z

> Use the results of Ex. 5.11 to find the magnetic field inside a solid sphere, of uniform charge density ρ and radius R, that is rotating at a constant angular velocity ω.

> (a) Check product rule (iv) (by calculating each term separately) for the functions A = x xˆ + 2y yˆ + 3z zˆ; B = 3y xˆ − 2x yˆ. (b) Do the same for product rule (ii). (c) Do the same for rule (vi).

> In 1897, J. J. Thomson “discovered” the electron by measuring the charge-to-mass ratio of “cathode rays” (actually, streams of electrons, with charge q and mass m) as follows: (a) First he passed the beam through uniform crossed electric and magnetic fiel

> Suppose you want to de&iuml;&not;&#129;ne a magnetic scalar potential U (Eq. 5.67) in the vicinity of a current-carrying wire. First of all, you must stay away from the wire itself / but that&acirc;&#128;&#153;s not enough. Show, by applying Amp&Atilde;

> (a) Check that Eq. 5.65 is consistent with Eq. 5.63, by applying the divergence. (b) Check that Eq. 5.65 is consistent with Eq. 5.47, by applying the curl. (c) Check that Eq. 5.65 is consistent with Eq. 5.64, by applying the Laplacian.

> Find the vector potential above and below the plane surface current in Ex. 5.8.

> (a) By whatever means you can think of (short of looking it up), find the vector potential a distance s from an infinite straight wire carrying a current I . Check that ∇ · A = 0 and ∇ × A = B. (b) Find the magnetic potential inside the wire, if it has rad

2.99

See Answer